can someone help me solve this?

Can Someone Help Me Solve This?

Answers

Answer 1

Answer:

- 9

Step-by-step explanation:

Given:

3q + 4 + 9 = - 14

Solving in steps:

3q + 13 = - 143q = - 14 - 133q = - 27q = - 27/3q = - 9
Answer 2

[tex]\\ \sf\longmapsto 3q+4+9=-14[/tex]

[tex]\\ \sf\longmapsto 3q+13=-14[/tex]

[tex]\\ \sf\longmapsto 3q=-14-13[/tex]

[tex]\\ \sf\longmapsto 3q=-27[/tex]

[tex]\\ \sf\longmapsto q=-9[/tex]


Related Questions

Pizza Palace sells cheese pepperoni, sausage and mushroom pizzas. Caleb noticed that for every 10 orders of cheese pizza, there were 7 orders of pepperoni pizza, 5 orders of sausage pizza and 8 orders of mushrooms pizza. If there were 35 pepperoni pizzas ordered, how many total pizzas were ordered?

Answers

Step-by-step explanation:

cheese : pepperoni : sausage : mushroom =

10 : 7 : 5 : 8

so, in total there are 10+7+5+8 = 30 "business pieces".

to get the total number of pizzas, we need to see how many pizzas are sold per "business piece" ?

we get this from the 35 orders of pepperoni pizzas.

these 35 orders represent 7 "business pieces".

that means 1 "business piece" = 35/7 = 5 pizzas.

that means that 30 "business pieces" are 5×30 = 150

so, in total, 150 pizzas were ordered.

A random number generator picks a number from 6 to 66 in a uniform manner. Round answers to 4 decimal places when possible.


The mean of this distribution is =

The standard deviation is =

The probability that the number will be exactly 19 is P(x = 19) =

The probability that the number will be between 28 and 46 is P(28 < x < 46) =

The probability that the number will be larger than 48 is P(x > 48) =

P(x > 17 | x < 62) =

Find the 49th percentile =

Find the maximum for the lower quartile =

Answers

Answer:last one

Step-by-step explanation:

A random number generator picks a number from 6 to 66 in a uniform manner, its resultants are  mathematically given as

X=36[tex]\sigma=5[/tex]p(x=19)=0.2167P(28 < x < 46) =0.3 P(x > 48)=0.3P(x > 17 | x < 62) =0.75k=29.4h=15

What is the mean of this distribution?

Generally, the equation for Mean is mathematically given as

[tex]X=\frac{a+b}{2}\\\\Therefore\\\\X=\frac{6+66}{2}[/tex]

X=36

b)

Generally, the equation for standard deviation is mathematically given as

[tex]\sigma=\sqrt{\frac{b-a}{12}^2}\\\\Therefore\\\\\sigma=\sqrt{(\frac{66-6}{12})^2}\\\\\sigma=\sqrt{25}\\\\[/tex]

[tex]\sigma=5[/tex]

c)

The probability that the number will be exactly 19

[tex]p(x=19)=\frac{19-6}{66-6}[/tex]

p(x=19)=0.2167

d)

The probability that the number will be between 28 and 46 is P(28 < x < 46)

[tex]P(28 < x < 46) =\frac{46-6}{66-6} -(\frac{28-6}{66-6})[/tex]

P(28 < x < 46) =0.3

e)

The probability that the number will be larger than 48 is P(x > 48)

[tex]P(x > 48)=1-P(x\leq 48)[/tex]

Therefore

[tex]P(x > 48)=1-(\frac{48-6}{66-6})[/tex]

P(x > 48)=0.3

f)

[tex]P(x > 17 | x < 62) = P(x\leq 62-P(x leq 17))[/tex]

Therefore

[tex]P(x > 17 | x < 62) =\frac{62-6}{66-6}-(\frac{17-6}{66-6})[/tex]

P(x > 17 | x < 62) =0.75

g)

The 49th percentile

[tex]p(x\leqk=k.(1/66-6))[/tex]

Therefore

[tex]0.49=k*(1/(66-6))[/tex]

k=29.4

h)

The maximum for the lower quartile

P(xl)=h*1/60

0.25=h*1/60

h=15

Read more about probability

https://brainly.com/question/795909

#SPJ6

(x-4)2 = 36
Which of the following expressions represents the solutions to the given equation?
0-4+ V(36)
O4 + V(36)
16+ (36)

Answers

There is 2 equation or 2 x if you prefer

1. X-4= 6. That is equal to 10 so x1=10
2. X-4= -6. That is equal to -2 so x2= -2

workout the size of angle x

Answers

Answer:

Where is it?

Step-by-step explanation:

There are 150 buttons in a box.
24 children take 3 buttons each.
How many buttons are left in the box?
Show your method.

Answers

24 times 3 is 72 then subtract 72 from 150 which is 78

The answer is 78 :)

Answer:

78 buttons left

Step-by-step explanation:

I multiplied 24 children by 3 buttons because each child took 3 buttons and I got 72 buttons taken.

Then I subtracted 150 buttons which is the starting number by 72 buttons (the amount taken by the child's) to get 78 buttons left in the box. ( I just separated negative 72 buttons into -50 and -22 to make it easier)

At a local print shop, 14 copies can be made for $4. At this rate, how how many copies could be made for $8?

Full explanation (No links)

Answers

Answer:

28

Step-by-step explanation:

the ratio is written as copies/dollars.

14/4=x/8

since we can multiply 4 by 2 to get 8, multiply 14 by 2 as well.

14/4=28/8

Ok so the answer is 28 copies for $8



The reason that’s correct is because $8 is 2 times more than $4. So we would do 14 (which is the amount of copies that are made for $4) x 2 = 28 copies

find the value of the angles marked 'x' and 'y' ​

Answers

Answer:

x=120 y=60

Step-by-step explanation:

Interior angle of a hexagon = 120

Straight line = 180

180-120=60

If $20,000 is invested at 7% for 15 years, find the future value if the interest is compounded daily(365 days)

Answers

Answer:

The total compound interest is $37,147.24.

Step-by-step explanation:

I could be wrong

what is 0.5734 rounded to the nearest thousandth​

Answers

Answer:

0.573

Hope that helps :)

0.5734 rounded to the nearest thousandth is 0.573

Explanation:

3 is the thousandths, so you would round up by looking at the number to the right which is 4. Since 4 is not greater than 5, it wouldn’t change the 3.

Joe cuts ten pizzas into quarters. How many pizza quarters are there in total?

Answers

Answer:

40 pizzas in total

Step-by-step explanation:

10×4=40

Kathy is following a recipe for punch that calls for 3 liters
of juice. She has 3 quarts of juice. Without using an equation, does Kathy
have enough juice for the punch? Explain.

Answers

Answer:

Yes, Kathy does  have enough juice.

Step-by-step explanation:

Kathy does have enough because quarts are a little bigger than liters, so the 3 quarts she has will cover the liters.

Do you think you could use a multiply a 4 digit number by a 1 digit number

Answers

Answer:

line them up straight with the bottom number and the top number.

A parabola having its axis vertical passes through three points A(0,0), B(1,0), and C(5,-20) respectively. Determine the coordinates of the vertex of the parabola.

Answers

Answer:

(1/2, 1/4)

Step-by-step explanation:

This is gonna be long. The x coordinate of a parabola's vertex = -b/2a. Now we have to find the equation of the parabola. Using the given points and plugging them into y=ax^2 + bx + c, we can determine:

0 = c

0 = a + b

-20 = 25a + 5b

After rearranging and solving, we get:

b = -a

-20 = 20a

a = -1

b = 1

y = -x^2 + x

Now we can find the x coordinate of the vertex, which is 1/2. After plugging this into our parabola, we get a y coordinate of -1/4 + 1/2 = 1/4. Therefore, the vertex is located at (1/2, 1/4).

HELLP ILL GIVE BRSINLY

Answers

m = 180 - CBE = 180 - [ 360 - BDC - BED - (180 - DCE - DEC)]  = 180 - [ 360 - 90 -90 - (180 - 13*2)] = 154

P/s: 180 - DCE - DEC = CDE

ok done. Thank to me :>

Louis is comparing the populations of several nearby cities. The population of Liberty Crossing can be written as 49 people, and the population of Harrisburg can be written as 47 people. The population of Glenview is 43 times the population of Harrisburg. Drag the cities to put them in order from least population (top) to greatest population (bottom).

Answers

Step-by-step explanation:

Louis is comparing the populations of several nearby cities. The population of Liberty Crossing can be written as 49 people, and the population of Harrisburg can be written as 47 people. The population of Glenview is 43 times the population of Harrisburg. Drag the cities to put them in order from least population (top) to greatest population (bottom).

The order of the cities from least population to greatest population:

Harrisburg

Glenview

Liberty Crossing

Given the information provided:

Harrisburg: This city has a population of 47 people. It's the smallest population among the three cities.

Glenview: The population of Glenview is 43 times the population of Harrisburg.

Since Harrisburg has 47 people, the population of Glenview would be 43 x 47 = 2021 people.

So, Glenview has a larger population than Harrisburg but is still smaller than Liberty Crossing.

Liberty Crossing: The population of Liberty Crossing is given as 49 people, which is the largest population among the three cities.

So, the correct order from least population to greatest population is:

Harrisburg

Glenview

Liberty Crossing

Learn more about Ascending order click;

https://brainly.com/question/31946606

#SPJ3

scientific form of 47500000000​

Answers

[tex]▪▪▪▪▪▪▪▪▪▪▪▪▪  {\huge\mathfrak{Answer}}▪▪▪▪▪▪▪▪▪▪▪▪▪▪[/tex]

Scientific notation for above value is ~

[tex]4.75 \times 10 {}^{10} [/tex]

[tex]Hello[/tex] [tex]There![/tex]

First, let's make sure our coefficient is between 1 and 10:

4.75

All numbers in scientific notation look like this:

___x10^- (some number)

4.75x10^

Now, the number is large, so the exponent is positive.

So we move the decimal place some number of units. We move the decimal place 10 units, so the exponent is 10:

4.75x10^10

Hope this helps!

~Just a felicitous girlie

#CarryOnLearning

[tex]SilentNature[/tex]

What is the image of the point (1,-4) after a rotation of 180° counterclockwise
about the origin?

Answers

Answer:

  (-1, 4)

Step-by-step explanation:

Rotation 180° negates both coordinates:

  (x, y) ⇒ (-x, -y)

The direction (clockwise or counterclockwise) is irrelevant. It is the same as reflection across the center of rotation (the origin).

  (1, -4) ⇒ (-1, 4) . . . . after rotation 180°

can you please help out my daughter on this...​

Answers

Answer:

d

Step-by-step explanation:

I’m like 80% sure that the answer is c.

Timmy has a total of 15 dimes and nickles i his pocket. He has 7 more dimes than nickels. How many dimes does he have.

Answers

Answer:

1.98 I think sorry if it's wrong

Can some help me please I did one but I am not sure if it is correct.
thank you

Answers

Answer:

yes its correct

Step-by-step explanation:

but for the check where did you get the 12? you might want to redo that part but otherwise its right

pls helpppppp i begggg​

Answers

Answer

x Is less than 4

Step-by-step explanation:

It will be an open circle and the arrow is going the left direction on the number 4

Can Someone help me on this math question please
I will mark u brainless please ​

Answers

it's easy just use bidmas

Help help help
Help hep help

Answers

Answer:

-2

Step-by-step explanation:

-4x-2x ≤ 17-5

-6x ≤ 12 | /(-6)

x ≥ -2

Find the area of a rectangle with sides measurements 8 inches and 17 inches

Answers

Answer:

136 inches or 354.44cm

Step-by-step explanation:

17*8 = 136

Answer:

136 inches

Step-by-step explanation:

17 X 8 = 136 inches

For a weekly car rental, a car rental company charges customers a flat fee of $100 plus
10 cents for every mile traveled.
15 Write an equation for the amount, A dollars, of a weekly rental for a car that travels
m miles

Answers

The way you typed out this sentence was a bit confusing but I think you were saying...

For a weekly car rental, a car rental company charges customers a flat fee of $100 +  and charges 10 cents for every mile traveled. write an equation for the amount of a weekly rental car that travels 15 miles (Please let me know if I am wrong)

The equation would be

10+10 +10+ 10+ 10+ 10+ 10+ 10+ 10+ 10+ 10+ 10+ 10+ 10+ 10 = 150 cents

so they would be charging $1.50

Hoped this helped!

Have a great day/night! :)

Can anyone help me with this one asap? Thanks in advance.

Answers

The answer to your question is in the image above.

rewrite 12/12 as a whole number

Answers

9514 1404 393

Answer:

  1

Step-by-step explanation:

Any (non-zero) number divided by itself is 1.

_____

This is a fact that has many uses in problem solving and units conversion.

what is 2x-4y=12 x-3y=17

Answers

Answer:

x=-16

y=-11

Explanation:

2x-4y=12

x-3y=17

first multiply all the numbers of the bottom equation by -2

2x-4y=12

-2x+6y=-34

then cancel the x's, and add the other numbers

2y=-22

solve for y

y=-11

plug y into either the original top or bottom equations

2x-4(-11)=12

2x+44=12

2x=-32

x=-16

or

x-3(-11)=17

x+33=17

x=-16

Write a system of equations to describe the situation below, solve using any method, and fill in the blanks.

A pair of kids and a pair of adults decided to compete in a three-legged race. The kids got to start 15 yards ahead of the adults, since they had shorter legs. When they were told to start, the kids hobbled forward at a rate of 1 yard per second, and the adults hobbled after them at a rate of 4 yards per second. Soon they were side-by-side. How long did that take? How far did the adults go?


It took seconds for the adults to go yards and catch up to the kids?

Answers

A system of equations is simply a system that contains related equations of multiply variables

It took 5 seconds for the adults to go 20 yards and catch up to the kids

Let x represents the time of the race

The kids started 15 yards ahead, and they raced at 1 yard per second.

So, the kids' equation is represented as: [tex]\mathbf{k =15 + x}[/tex]

The adults' raced at 4 yards per second.

So, the adults' equation is represented as: [tex]\mathbf{a =4x }[/tex]

When they are side by side, we have the following equivalent equation

[tex]\mathbf{4x = 15 + x}[/tex]

Subtract x from both sides

[tex]\mathbf{3x = 15}[/tex]

Divide both sides by 3

[tex]\mathbf{x = 5}[/tex]

It means that, it took the adults 5 seconds to catch up with the kids

Substitute 5 for x in any of the equations to calculate the distance travelled.

[tex]\mathbf{a =4 \times 5}[/tex]

[tex]\mathbf{a =20}[/tex]

It means that, the adults catch up with the kids at 20 yards

Read more about systems of equations at:

https://brainly.com/question/12895249

Answer:

The adults went 123 yards and caught up to the kids in 41 seconds

Step-by-step explanation:

ixl :)

there are 350,000 in a warehouse predict the number of light bulbs that are likely to have no defects

Answers

Answer:

175000.(or around that number)

Step-by-step explanation:

half of 350,000 is 175000.

Other Questions
2x+3y=15x+y=6work out the values of x and y. Leah bought 3/4 pound of trail mix to make snack bags. She put 1/8 pound of trail mix into each snack bag. Which expression can be used to find how many snack bags Leah made? (Show work and simplify pls) if 15 grams of carbon is present at the beginning of a reaction,how many grams of carbon will be present in the products? Which if the following statements most directly expresses the authors thesis in the passage? Tayla purchased a large box of comic books for $300. She gave 15 of the comic books to her brother and then sold the rest on an Internet website for $330, making a profit of $1.50 on each one. How many comic books were in the box AgNO3(aq) + NaCl (aq) > AgCl (s) + NaNO3 (aq)The equation represents a __ reaction.A. synthesis B. decomposition C. neutralization D. double replacement What was the role of judicial review in FBI versus fazagaThe case is about the fbi illegally spying on a Muslim community in California what is one benefit of utilizing social media to deliver advertising messages? How do we pay homage to the martyrs? before the united states how many colonies were there Find the volume of the composite figure.Write with the correct unit of measurement, such as 5 cm^2. After reading the first amendment, explain all the personal freedoms it protects, while also making an argument why or why not someone should stand and salute in respect of the American Flag. The ancient patriarchal system of our country refers to theory seeks to explain how we decide, on the basis of samples of an individual's behavior, what are the specific underlying causes of that person's behavior. Jasmine got a new puppy for her birthday. He's full of energy, so she takes him for a walk along a 1.2-mile loop in a nearby park. If they walked a total of 3.6 miles, how many loops did they do? Tama o maliplease answer this Q.5) The graph shows the number of inches a plant grew each week. Between which 2 weeks did the plant didn't grow at all?Week 1 and Week 2Week 4 and Week 5Week 2 and Week 3 Select the correct answer.Solve the following inequality.-8 < 3x + 1 < 7A. B. C. D. On a coordinate plane how are the location of points -6,7 and -6,-7 related How does the author use paragraphs 30-31 to refine their ideas? Cite evidence in your answer. Commonlit- DECLARATION OF INDEPENDENCEWill give brainly if well written!